LSAT and Law School Admissions Forum

Get expert LSAT preparation and law school admissions advice from PowerScore Test Preparation.

User avatar
 Dave Killoran
PowerScore Staff
  • PowerScore Staff
  • Posts: 5853
  • Joined: Mar 25, 2011
|
#92229
Complete Question Explanation
(The complete setup for this game can be found here: lsat/viewtopic.php?f=371&t=15040)

The correct answer choice is (C).

This can be a difficult question. From a theoretical standpoint, it would have been preferable if the question #9 preceded this question since ultimately the two question are quite similar, and the theory behind question #7 is a more complex version of the theory behind question #9. There are really two ways to solve this question: abstractly or by using hypotheticals. We will examine both methods.

Abstract Solution

Rule: H is assigned only once per the question stem.
Rule: F cannot serve on both the Quality and Sales committees (due to the local FM rule) and G cannot serve on both the Quality and Sales      committees (due to the global GL rule).
Inference: G serves once on either the Quality or Sales committee, and F serves once on either the Quality or Sales committee.
Inference: Since G must be assigned to either the Quality or Sales committee, L is assigned to the committee G is not on, and since F must be      assigned to either the Quality or Sales committee, M is assigned to the committee G is not on.
Inference: H, G, L, F, and M have now been assigned to their maximum number of spaces, and one space still remains on the Quality Committee and      the Sales Committee.
Inference: K must fill those final two spaces, and thus K is assigned to exactly two committees.
Essentially, K Hurdles the Uncertainty and must be assigned to both committees.

Hypotheticals

Only two hypotheticals exist for this question:

G2-Q7-d1.png

Accordingly, answer choice (C) is correct.
You do not have the required permissions to view the files attached to this post.
 DAthenour
  • Posts: 16
  • Joined: Sep 21, 2017
|
#40182
Hi PowerScore team,

I was wondering if you could help walk me through how to do the hypotheticals required for number 7 in a way that doesn't eat up the clock. This question took me a really long time in my game practice, and I'm not sure if there's an easier way of going about it other than trying the two following combinations:

F K M
G L K
H F G

and

H K L
G M K
F G F

I understand why then the answer is C, but this seems like it takes a lot of time and effort. Thank you for your help!!
 Jennifer Janowsky
PowerScore Staff
  • PowerScore Staff
  • Posts: 90
  • Joined: Aug 20, 2017
|
#40188
Hi! Question 7 may be the toughest of this set, so it being second definitely warrants skipping it and coming back later. It isn't worth taking all your time for a new rule question. That being said, the quickest way to solve it would be as follows:

Initial Setup:

P Q S
F _ _
G _ _
H _ K


Since there can only be 1 H, that leaves only F/G as supervisors of the other groups:

P Q S
F F/G F/G
G _ _
H _ K


However, you cannot put F all the way across, becausewith G there must not be L, and with F there must not be M (new rule, can't be same committee) and having either of the same would make it impossible to place L or M. Therefore, there can only be an F and G in those places:

P Q S
F G F
G _ _
H _ K


Or

P Q S
F F G
G _ _
H _ K


This uses up all of the officers, leaving only supervisors. The supervisors must now be placed, but with G there must not be L, and with F there must not be M. Therefore, you cannot put either two M or two L across for the same reason--they would prevent either F or G from being on those committees. In that case, you have to switch them in order to place them:

P Q S
F G F
G M L
H _ K


Or

P Q S
F F G
G L M
H _ K


Finally, K must be placed in the last spot for the same reason: putting G would prevent L, L would prevent G, M would prevent F, and F would prevent M. This leaves:

P Q S
F F G
G L M[/b]
H K K


Therefore it must be true that K is on 2 committees.

Overall, still a long and tough problem to solve, and you may be right to skip it and return later. Hopefully this was helpful!
 DAthenour
  • Posts: 16
  • Joined: Sep 21, 2017
|
#40197
This was very helpful thank you so much!
 saygracealways
  • Posts: 34
  • Joined: Apr 09, 2020
|
#75616
Hi PowerScore,

I did answer this question correctly while solving the game under timed conditions, but I'm still struggling to understand why K must be assigned to exactly 2 committees?

Please refer to the hypotheticals I drew below for reference, but basically I thought that even when G and M are assigned to Quality and F K L are assigned to Sales, the last remaining employee assigned to Quality could be either M or K. Alternatively, even when F and L are assigned to Quality and G K M are assigned to Sales, I thought the last remaining employee assigned to Quality could be either L or K. This doesn't seem to violate neither the numerical distribution of 2-2-2-1-1-1 nor any of the global/local rules, so I'm struggling to understand why K must be assigned to both Quality and Sales? Would that also be a Not Necessarily True answer because K could possibly be assigned to just Sales (1 committee)?


Please let me know where my reasoning is going wrong here, thank you so much!
You do not have the required permissions to view the files attached to this post.
 Adam Tyson
PowerScore Staff
  • PowerScore Staff
  • Posts: 5153
  • Joined: Apr 14, 2011
|
#75960
Hey saygrace, it looks like you are overlooking the fact that each committee must have three employees assigned to it. An assignment of GMM violates that, because M is still just one person, and that committee only has G and M on it! You can't use the same person twice and claim you have three people on the committee! Same problem with an FLL assignment, of course - you can't count L twice like that.
User avatar
 Esquire123
  • Posts: 15
  • Joined: Jan 25, 2023
|
#99346
The last rule, “K must be assigned to the Sales Committee has me really confused. “Must” is a group two conditional indicator, so it introduces a necessary condition. Written into logic, the last rule looks like: K—->Sales

With that, I’m confused with how K was able to be placed in a location other than Sales (specific to question 7)
User avatar
 Jeff Wren
PowerScore Staff
  • PowerScore Staff
  • Posts: 389
  • Joined: Oct 19, 2022
|
#99354
Hi Esquire123,

The final rule is actually not conditional in nature. In other words, it is not stating "If K is selected, then it must go on the Sales Committee," or "K can only be assigned to the Sales Committee."

The rule is simply saying in effect that "K is assigned to the Sales Committee." Because employees can be on more than one committee in this game (and some have to be to fill all of the spots), the fact that K is on the Sales Committee doesn't mean that K cannot serve on another committee.

While the word "Must" is a necessary indicator word, meaning that the word can be used conditionally, it isn't always used conditionally. Similarly, words like "if" and "only" are not always used conditionally, so you need to read and interpret the meaning of each sentence in context.

Here "must" is just being used to convey certainty. It means that K is definitely assigned to the Sales Committee, although it can also be assigned elsewhere.

Get the most out of your LSAT Prep Plus subscription.

Analyze and track your performance with our Testing and Analytics Package.